Can you solve this?

Number Theory Level pending

Find number of pairs of positive integers m , n 3 m,n\geq3 for which there exist infinitely many positive integers a such that a m + a 1 a n + a 2 1 \frac{a^m+a-1}{a^n+a^2-1} is itself an integer.


The answer is 1.

This section requires Javascript.
You are seeing this because something didn't load right. We suggest you, (a) try refreshing the page, (b) enabling javascript if it is disabled on your browser and, finally, (c) loading the non-javascript version of this page . We're sorry about the hassle.

1 solution

Suppose that we can write a m + a 1 = ( a n + a 2 1 ) Q ( x ) + R ( x ) a^m+a-1=(a^n+a^2-1)Q(x)+R(x) , with R ( x ) R(x) not identically zero. Then by the conditions, R ( x ) R(x) must have infinitely many roots, contradiction; hence, a n + a 2 1 a m + a 1 a^n+a^2-1|a^m+a-1 . Since 0 n + 0 2 1 < 0 0^n+0^2-1<0 and 1 n + 1 2 1 > 0 , a n + a 2 1 = 0 1^n+1^2-1>0, a^n+a^2-1=0 has a root 0 < r < 1 0<r<1 . Then also r m + r 1 = 0 r^m+r-1=0 , so r m + r = r n + r 2 r ( 1 r ) = r n ( 1 r m n ) r ( 1 r ) = ( 1 r 2 ) ( 1 r m n ) r^m+r=r^n+r^2 \Rightarrow r(1-r)=r^n(1-r^{m-n}) \Rightarrow r(1-r)=(1-r^2)(1-r^{m-n}) . Continuing, we have that r = 1 r m n + r r m n + 1 r m n + 1 + r m n = 1 r=1-r^{m-n}+r-r^{m-n+1} \Rightarrow r^{m-n+1}+r^{m-n}=1 . Since r < 1 r<1 , we must have m n + 1 n m-n+1\le n or else r m n + 1 + r m n < r n + r n 1 r n + r 2 = 1 r^{m-n+1}+r^{m-n}< r^n+r^{n-1}\le r^n+r^2=1 , contradiction. Hence, m 2 n 1 m\le 2n-1 .

Clearly, a n + a 2 1 ( a m n ( a n + a 2 1 ) ( a m + a 1 ) ) a n + a 2 1 a m n + 2 a m n a + 1 a^n+a^2-1|(a^{m-n}(a^n+a^2-1)-(a^m+a-1)) \Rightarrow a^n+a^2-1|a^{m-n+2}-a^{m-n}-a+1 . Since a m n + 2 a m n a + 1 a^{m-n+2}-a^{m-n}-a+1 is not identically zero, and since m 2 n 1 , m n + 2 m\le 2n-1, m-n+2 is either m m or m + 1 m+1 . In the first case, we must have a m n + 2 a m n a + 1 = a n + a 2 1 a^{m-n+2}-a^{m-n}-a+1=a^n+a^2-1 , but this is impossible. In the second case, we must have ( a 1 ) ( a n + a 2 1 ) = a n + 1 a n 1 a + 1 a n 1 = a n + a 3 a 2 (a-1)(a^n+a^2-1)= a^{n+1}-a^{n-1}-a+1 \Rightarrow -a^{n-1}=-a^n+a^3-a^2 , which can only happen if n = 3 n=3 , and therefore m = 5 m=5 . As ( a 3 + a 2 1 ) ( a 2 a + 1 ) = ( a 5 + a 1 ) (a^3+a^2-1)(a^2-a+1)=(a^5+a-1) , this solution works. So the only answer is ( m , n ) = ( 5 , 3 ) (m,n)=(5,3) .

This is IMO 2002/3 (Thanks Mihir for correcting!)

Calvin Lin Staff - 6 years, 4 months ago

Log in to reply

IMO 2002/p3

mihir Chakravarti - 6 years, 4 months ago

You were at the 2002 IMO, right?

mihir Chakravarti - 6 years, 4 months ago

Log in to reply

Yup I was. it was either 2001/2002 and was the 3rd question of the day, so I randomly guessed. Guessed both choices wrongly lol.

Calvin Lin Staff - 6 years, 4 months ago

0 pending reports

×

Problem Loading...

Note Loading...

Set Loading...